LSAT and Law School Admissions Forum

Get expert LSAT preparation and law school admissions advice from PowerScore Test Preparation.

 abajaj
  • Posts: 20
  • Joined: Sep 20, 2012
|
#6258
Hi,

I am currently working on the LR Problem Set #1 and have the following question:

#38: This is one that I just can't seem to understand. The diagram I drew for this was the following:

Generous Parents :most: Good Parents :arrow: Good listeners
Self-Centered Parents :some: Good Parents :arrow: Good Listeners

I chose answer B because I ruled out A and E first, and then thought that C and D was mistaken reversals. I'm not quite sure why the answer is D.

Any help would be appreciated, thank you so much!!!
User avatar
 Dave Killoran
PowerScore Staff
  • PowerScore Staff
  • Posts: 6030
  • Joined: Mar 25, 2011
|
#6260
Hi Aba,

Answer choice (D) comes directly from this relationship that you drew:

..... Self-Centered Parents :some: Good Parents :arrow: Good Listeners

You have the diagram correct, so it looks like you may have just overlooked the some relationship here. That relationship produces the inference that:

..... ..... ..... ..... ..... SC :some: GL

"Some" reads both ways, and for a refresher on that, check out the Lesson 8 Homework Supplements area online. There's extensive coverage of Formal Logic there.

Answer choice (B) is incorrect because Good Parents group and the Good Listeners group could be identical, and thus you would not know that some parents who are good listeners are not good parents.

With (C) and (D), there are no Mistaken Reversals with "some" and "most" statements--they don't exist (answer choice (A) is actually the MR here).

Please let me know if that helps. Thanks!
 abajaj
  • Posts: 20
  • Joined: Sep 20, 2012
|
#6267
This explanation definitely helps a lot, thank you! And I will be checking out the homework supplement for sure tonight!

Get the most out of your LSAT Prep Plus subscription.

Analyze and track your performance with our Testing and Analytics Package.